ChaseDream
搜索
返回列表 发新帖
楼主: sdcar2010

SDCAR2010【逻辑入门】(八)Assumptions

[精华] [复制链接]
发表于 2011-7-9 16:46:14 | 显示全部楼层
这一部分真是我的软肋啊,看这篇帖子都比较吃力。。。
发表于 2011-7-15 22:15:12 | 显示全部楼层
牛牛,sufficient和necessary分不清啊,牛牛给的例子看得很清楚,不过出在题目里面就分不清啦,感觉justify the conclusion 和 rely on the assumption差不多啊,求解答~
发表于 2011-7-15 23:25:01 | 显示全部楼层
For question No.2
2. A researcher discovered that people who have low levels of immune-system activity tend to score much lower on tests of mental health than do people with normal or high immune-system activity. The researcher concluded from this experiment that the immune system protects against mental illness as well as against physical disease.
The researcher's conclusion depends on which of the following assumptions?
(A) High immune-system activity protects against mental illness better than normal immune system activity does.
(B) Mental illness is similar to physical disease in its effects on body systems.
(C) People with high immune-system activity cannot develop mental illness.
(D) Mental illness does not cause people's immune system activity to decrease.
(E) Psychological treatment of mental illness is not as effective as is medical treatment.

But if you negate C. people with high immune-system activity CAN develop mental illness. If that is true, then the conclusion that the immune system protects against mental illness is wrong. So why choose D but not c?
 楼主| 发表于 2011-7-16 00:02:26 | 显示全部楼层
For question No.2
2. A researcher discovered that people who have low levels of immune-system activity tend to score much lower on tests of mental health than do people with normal or high immune-system activity. The researcher concluded from this experiment that the immune system protects against mental illness as well as against physical disease.
The researcher's conclusion depends on which of the following assumptions?
(A) High immune-system activity protects against mental illness better than normal immune system activity does.
(B) Mental illness is similar to physical disease in its effects on body systems.
(C) People with high immune-system activity cannot develop mental illness.
(D) Mental illness does not cause people's immune system activity to decrease.
(E) Psychological treatment of mental illness is not as effective as is medical treatment.

But if you negate C. people with high immune-system activity CAN develop mental illness. If that is true, then the conclusion that the immune system protects against mental illness is wrong. So why choose D but not c?
-- by 会员 glendasc (2011/7/15 23:25:01)



If you negate C), the conclusion still holds since the conclusion is "the immune system PROTECTs against mental illness,", which means people with high immune system have some protection but that protection is not bullet-proof. So people with high immune system could become mentally unstable, but less likely than those who have low immune system.
 楼主| 发表于 2011-7-16 00:03:39 | 显示全部楼层
牛牛,sufficient和necessary分不清啊,牛牛给的例子看得很清楚,不过出在题目里面就分不清啦,感觉justify the conclusion 和 rely on the assumption差不多啊,求解答~
-- by 会员 dualtunnal (2011/7/15 22:15:12)


justify the conclusion: sufficient assumption

rely on the assumption: necessary assumption
发表于 2011-7-21 13:39:12 | 显示全部楼层
For question No.2
2. A researcher discovered that people who have low levels of immune-system activity tend to score much lower on tests of mental health than do people with normal or high immune-system activity. The researcher concluded from this experiment that the immune system protects against mental illness as well as against physical disease.
The researcher's conclusion depends on which of the following assumptions?
(A) High immune-system activity protects against mental illness better than normal immune system activity does.
(B) Mental illness is similar to physical disease in its effects on body systems.
(C) People with high immune-system activity cannot develop mental illness.
(D) Mental illness does not cause people's immune system activity to decrease.
(E) Psychological treatment of mental illness is not as effective as is medical treatment.

But if you negate C. people with high immune-system activity CAN develop mental illness. If that is true, then the conclusion that the immune system protects against mental illness is wrong. So why choose D but not c?
-- by 会员 glendasc (2011/7/15 23:25:01)




If you negate C), the conclusion still holds since the conclusion is "the immune system PROTECTs against mental illness,", which means people with high immune system have some protection but that protection is not bullet-proof. So people with high immune system could become mentally unstable, but less likely than those who have low immune system.
-- by 会员 sdcar2010 (2011/7/16 0:02:26)

Dear scdcar2010,
Can you tell me how to negate this sentence "children who don't spend all of their after-school hours playing computer games spend at least some of that time talking with other people." Can i translate it into: children who don't spend all of their after-school hours playing computer games spend most of that time talking with other people?
Besides, how to use a negated sentence to undermine the conclusion, is it need to corporate the existed premises to undermine it?
 楼主| 发表于 2011-7-21 20:24:03 | 显示全部楼层
children who don't spend all of their after-school hours playing computer games spend NONE of that time talking with other people.
 楼主| 发表于 2011-7-21 20:24:36 | 显示全部楼层
children who don't spend all of their after-school hours playing computer games DO NOT spend at least some of that time talking with other people
发表于 2011-7-21 22:49:41 | 显示全部楼层
sdcar,可以转帖吗?更多人可以学习到~
发表于 2011-7-22 00:40:50 | 显示全部楼层
...
您需要登录后才可以回帖 登录 | 立即注册

Mark一下! 看一下! 顶楼主! 感谢分享! 快速回复:

手机版|ChaseDream|GMT+8, 2024-3-28 17:11
京公网安备11010202008513号 京ICP证101109号 京ICP备12012021号

ChaseDream 论坛

© 2003-2023 ChaseDream.com. All Rights Reserved.

返回顶部